LSAT and Law School Admissions Forum

Get expert LSAT preparation and law school admissions advice from PowerScore Test Preparation.

 Administrator
PowerScore Staff
  • PowerScore Staff
  • Posts: 8916
  • Joined: Feb 02, 2011
|
#33529
Complete Question Explanation

Flaw in the Reasoning—SN. The correct answer choice is (E)

The author of this stimulus presents a number of conditional statements, and draws a flawed conditional conclusion.
First, if the circulation of the martial arts magazine under discussion continues to increase as much as it has over the past ten years, in a decade it will be the best selling martial arts magazine in the world:
  • Premise (1): ..... Circ. increase at same rate ..... :arrow: ..... Best-selling magazine
However, the author goes on, the managing editor will not be allowed to make the changes that she wants, so the circulation will not continue to increase at the same rate as it has over the last ten years:
  • Premise (2): ..... Editor make changes ..... :arrow: ..... Circ. increase at same rate


Thus, the author concludes, in ten years from now, the magazine will not be the best selling martial arts magazine:
  • Conclusion: ..... Best-selling magazine
The statement above represents a classic conditional reasoning error: a Mistaken Negation. The question stem asks for the flaw in the author’s reasoning. Note that it can be very difficult to describe a conditional reasoning flaw without using the words “sufficient,” “necessary,” or a synonymous term.

Answer choice (A): The author does not conclude that no other changes are needed; rather, the author concludes that if the company cannot match its growth of the past ten years, becoming the best selling martial arts magazine in the world is impossible.

Answer choice (B): The author does not equate a reduced rate of increase with a decline; rather, the author concludes that a reduced rate of increase would preclude it’s becoming the best selling martial arts magazine in ten years’ time.

Answer choice (C): This incorrect answer choice describes circular reasoning, which is not the conditional reasoning flaw manifest in the stimulus.

Answer choice (D): This choice describes drawing an unwarranted conclusion on a single incompatibility. This is not the author’s flaw, which, as discussed, is a Mistaken Negation, so this choice should be ruled out of contention.

Answer choice (E): This is the correct answer choice, as it describes the Mistaken Negation reflected in the stimulus. Based on the fact that continuing to increase the circulation at the same rate would be sufficient to make the magazine the best selling in the world, the author concludes that such an increase is necessary (note that although this choice does not include the term “necessary,” it does include the synonymous term “required”).
 avengingangel
  • Posts: 275
  • Joined: Jun 14, 2016
|
#30933
It took me a minute or two to finally choose the correct answer choice, so, just to be sure of my reasoning...

I diagrammed (abbreviating the variables here for sake of easy reading/understanding):

Premise: circulation rise :arrow: largest-selling mag
Premise: circulation rise :arrow: changes
Conclusion: largest-selling mag

Since I was kind of thrown off by the single statement of the conclusion (rather than a conditional statement), I then thought it might help if I took the contrapositive of the 2nd premise (since the conclusion heavily relies on that relationship), giving me:

changes :arrow: circulation rise

After not really liking any of the answer choices, I looked at (E) and saw how it matched up with my diagram... The argument is taking changes, which is sufficient (or, ensuring -- right??) for a certain outcome (circulation rise) as something that is required for that outcome (again, circulation rise).

Is that all correct ??

And if so, could someone please explain it in more "human" terms? I was only able to get to this answer by converting the entire argument into a diagram and losing all connection to the actual content (which I know, is the main thing you need to be able to do to do well on the LSAT!) but I'm not really able to convert the diagram back to the content of the argument , at least in a way that makes sense to me... Like, if I try to think about it in real life terms (gasp!), then the flaw identified still doesn't make sense to me, and it actually seems like a valid argument..: if those changes aren't made, then you're not going to have the rise in circulation, which you need in order to be the largest selling magazine!?! Right ?!!!

UGHHHHGHGHG.. seee, I totally don't get this.. because I am realizing that that last part I just typed (the rise in circulation, which you need in order to be the largest selling magazine) is in fact not true, because it's saying largest selling mag :arrow: rise in circulation, which is the mistaken reversal of my first diagram, circulation rise :arrow: largest selling mag. But my mind right now won't even wrap my head around that.

Sorry for the free-flow of thought here, hope it's not too confusing. I guess my lesson here is just to diagram the dang thing and get on with it to the next question...

Thanks.
 David Boyle
PowerScore Staff
  • PowerScore Staff
  • Posts: 836
  • Joined: Jun 07, 2013
|
#31448
avengingangel wrote:It took me a minute or two to finally choose the correct answer choice, so, just to be sure of my reasoning...

I diagrammed (abbreviating the variables here for sake of easy reading/understanding):

Premise: circulation rise :arrow: largest-selling mag
Premise: circulation rise :arrow: changes
Conclusion: largest-selling mag

Since I was kind of thrown off by the single statement of the conclusion (rather than a conditional statement), I then thought it might help if I took the contrapositive of the 2nd premise (since the conclusion heavily relies on that relationship), giving me:

changes :arrow: circulation rise

After not really liking any of the answer choices, I looked at (E) and saw how it matched up with my diagram... The argument is taking changes, which is sufficient (or, ensuring -- right??) for a certain outcome (circulation rise) as something that is required for that outcome (again, circulation rise).

Is that all correct ??

And if so, could someone please explain it in more "human" terms? I was only able to get to this answer by converting the entire argument into a diagram and losing all connection to the actual content (which I know, is the main thing you need to be able to do to do well on the LSAT!) but I'm not really able to convert the diagram back to the content of the argument , at least in a way that makes sense to me... Like, if I try to think about it in real life terms (gasp!), then the flaw identified still doesn't make sense to me, and it actually seems like a valid argument..: if those changes aren't made, then you're not going to have the rise in circulation, which you need in order to be the largest selling magazine!?! Right ?!!!

UGHHHHGHGHG.. seee, I totally don't get this.. because I am realizing that that last part I just typed (the rise in circulation, which you need in order to be the largest selling magazine) is in fact not true, because it's saying largest selling mag :arrow: rise in circulation, which is the mistaken reversal of my first diagram, circulation rise :arrow: largest selling mag. But my mind right now won't even wrap my head around that.

Sorry for the free-flow of thought here, hope it's not too confusing. I guess my lesson here is just to diagram the dang thing and get on with it to the next question...

Thanks.

Hello avengingangel,

Human terms. --Rising circulation will make the biggest magazine. Then, the stimulus says, there isn't going to be rising circulation, so, there won't be the biggest magazine. That's a classic Mistaken Negation. I.e., something else (e.g., Donald Trump's endorsement) could be the "sufficient" factor that makes the magazine the biggest; so rising circulation isn't really required, which you seem to have figured out above.

Hope this helps,
David
 avengingangel
  • Posts: 275
  • Joined: Jun 14, 2016
|
#32509
Yes, that makes so much sense! I just re-visited this question and it seems painfully easy to figure out without a whole diagram... I believe I was just thinking about it wayyy too hard. It's a simple mistaken negation, as you said! Thanks.
User avatar
 pmuffley
  • Posts: 39
  • Joined: Sep 24, 2021
|
#92769
This one is confusing me for sure.

B: equates a reduction in the rate at which the magazine’s circulation is increasing with a decline in the magazine’s circulation

I chose B because the author only said that the circulation has to continue to rise over the next 10 years. With the new changes, it will now just not rise as quickly. But if it rises as it has then it can still become the best magazine.

Just because there is a reduction does not mean that it is still not increasing...just at a different rate. There is no definitive statement that says it has to increase at the "same rate" just increase as it has been increasing. In a different question, you guys would tell me that I shouldn't make too many assumptions behind what the author intends, and here I'm sure you will tell me to make this assumption.
 Robert Carroll
PowerScore Staff
  • PowerScore Staff
  • Posts: 1787
  • Joined: Dec 06, 2013
|
#92812
pmuffley,

All of your reasoning here just proves that answer choice (E) is correct. The author is nowhere saying, implying, assuming, or otherwise that the magazine circulation will decline at all. The author even says "the magazine’s circulation will not rise as quickly". Well, "rising as quickly" was the sufficient condition, in the first sentence, of the conditional. The author thinks that, if it will not rise as quickly as would be sufficient to make it the largest-selling martial arts magazine, then it won't become such. Nowhere in there is the author thinking its circulation will decline. The author thinks that falling short of a certain rate of growth will keep it from becoming the largest-selling martial arts magazine. If that rate of growth was 10% a year, the author would think that only growing by 9% a year would keep the necessary condition from happening. This is a Mistaken Negation.

You're right that not rising as quickly could still be rising, and that means the magazine still could become the world leader. But that's the author's mistake - the author thought that anything short of the previous rate, even a positive rate of growth, is not enough. So:

If it rises as quickly, it will lead the world.

It won't rise as quickly.

Therefore, it won't lead the world.

Robert Carroll

Get the most out of your LSAT Prep Plus subscription.

Analyze and track your performance with our Testing and Analytics Package.